Download as pdf or txt
Download as pdf or txt
You are on page 1of 66

LESSON 1: Matter and its Classification

Introduction

What is the importance of chemistry and why would you want to learn about it?
Everyone can and should understand basic chemistry, but it may be important for
you to take a course in chemistry (as a requirement of your course) or even make a
career out of it. It's important to understand chemistry if you are studying any of the
sciences because all of the sciences involve matter and the interactions between
types of matter.

The scope of chemistry is extremely broad, and it touches every aspect of our lives.
An iron gate rusting, a pandesal bread baking, the production in a laboratory of an
antibiotic, the ignition of an automobile, and the digesting of food all fall the realm
of chemistry. The key to understanding such diverse processes is comprehending
the fundamental nature of matter which is under the umbrella of chemistry. Now,
take – off on your first journey of demystifying chemistry! Have fun learning!

Learning Objectives
At the end of this topic, students will be able to:

 Describe the different states and properties of matter;


 Explain how matter undergoes changes; and
 Distinguish among elements, compounds, and mixtures.

Activating Prior Learning

You must have heard the word ‗matter‘ and has learned about it during you
elementary grade, as well as in your middle school. Your memory of it might be a
little fuzzy as it happened couples of years ago, thus, this simple activity might help
you remember some of the concepts of matter. Using the word ‗MATTER‘ as an
acronym, write something to describe what matter is all about.

It Matters!

M–
A–
T–
T–

~1~
E–
R–

Nice try! Now proceed to the next station and explore more about our lesson for
today.

Presentation of Contents

With the information you supplied in the first activity, try to see whether it coincides
with definitions and facts presented in the texts that follows. Read carefully and with
understanding.

Chemistry: The Study of Matter

Chemistry is the field of study concerned with


the characteristics, composition, and
transformations of matter. But, what is matter?
Matter is anything that has mass and occupies
space. The term mass refers to the amount of
matter present in a sample, and the amount
of space occupied by the sample is termed
as volume.

Are you holding something right now? If not, is


there an object near you? Try inspecting it.
Does it have mass? How much do you think it
weighs? Is it occupying a portion of space?
Does it have dimensions like length, width and
height or in short, volume? If your answers are
affirmative, then it definitely is a matter. Are
you a matter? What do you think? Source:https://i.pinimg.com/originals/57/2f/95/
572f9558fd867137f52f4aa19518cdf1.png

Physical States of Matter

Everything that is familiar to us in our daily lives - from the land we walk on, to the
water we drink and the air we breathe - is based upon the states of matter called
gases, liquids, and solids – the three classic phases. This classification is based on
whether its shape and volume are definite or indefinite.

~2~
A peso coin, for example, is a solid with definite shape and volume. It retains its
shape and volume whether it is placed in a large container or table top. A liquid,
like coffee drink, has indefinite shape and always takes the shape of its container.
Liquid has definite volume; a 200ml coffee only feels up the cup at 200ml volume. A
gas, on the other hand, has both indefinite shape and volume. A gaseous
substance always completely fills its container, adopting both the container‘s
volume and shape.

The state of matter observed for a particular


substance depends on its temperature, the
surrounding pressure, and the strength of the forces
holding its structural forces together. At the
temperatures and pressures normally encountered
on Earth, water is one of the new substances found
in all three states: solid ice, liquid water, and
gaseous steam.
Source: Google images

Aside from the three above – mentioned conventional states of matter, there are
two more known states – Plasma and Bose – Einstein Condensate. Solid, liquid, gas,
and plasma are naturally occurring states of matter, while Bose – Einstein
Condensate is man – made.

Plasma is not a common state of matter


here on Earth, but it may be the most
common state of matter in the universe,
according to the Jefferson Laboratory.
Stars are essentially superheated balls of
plasma. Plasma consists of highly charged
particles with extremely high kinetic
energy. The noble gases (helium, neon,
argon, krypton, xenon and radon) are
often used to make glowing signs by using
electricity to ionize them to the plasma
state. Source:https://www.google.com/url?sa=i&url=https%3A%2F%2Fwww.sli
deshare.net%2Ftirth1508%2Fstatesmattera

On the other hand, Bose-Einstein condensate (BEC) was created by scientists in


1995. Using a combination of lasers and magnets, Eric Cornell and Carl Weiman,
scientists at the Joint Institute for Lab Astrophysics (JILA) in Boulder, Colorado,
cooled a sample of rubidium to within a few degrees of absolute zero. At this
extremely low temperature, molecular motion comes very close to stopping. Since
there is almost no kinetic energy being transferred from one atom to another, the

~3~
atoms begin to clump together. There are no longer thousands of separate atoms,
just one "super atom."

In many cases, chemical substances may occur in different forms but exist in the
same physical state. These substances are called allotropes. This concept of
substances was proposed by Jons Jakob Berzelius in 1841. Examples include
graphite and diamond, the allotropes of carbon. Oxygen molecule and ozone, the
allotropes of Oxygen are other examples of allotropes

Going Through the Phases

Adding or removing energy from matter causes a physical change as matter


moves from one state to another. For example, adding thermal energy (heat) to
liquid water causes it to become steam or vapor (a gas). And removing energy
from liquid water causes it to become ice (a solid). Physical changes can also be
caused by motion and pressure. Below are the processes that involve changing the
state of matter:

Melting (Solid Liquid)

When heat is applied to a solid, its particles begin to vibrate faster and move farther
apart. When the substance reaches a certain combination of temperature and
pressure, its melting point, the solid will begin to melt and turn into a liquid. Example:
Heating a bar of butter in a pan to melt it.

Freezing (Liquid Solid)

When heat is removed from a liquid, its particles slow down and begin to settle in
one location within the substance. When the substance reaches a cool enough
temperature at a certain pressure, the freezing point, the liquid becomes a solid.
Example: Freezing a bag of water to make ice.

Sublimation (Solid Gas)


When a solid is converted directly into
a gas without going through a liquid
phase, the process is known as
sublimation. Example: Dry ice
(solidified carbon dioxide) sublimes to
gaseous CO2.
Vaporization (Liquid Gas)

Vaporization is the conversion of a


liquid to a gas and can occur through
either evaporation or boiling. Example:

~4~
You see steam (water vapor) arising from a kettle when its water boils.

06/cos.jpg

Condensation (Gas Liquid)

Condensation occurs when a gas loses energy and comes together to form a
liquid. For example, water vapour in the atmosphere condenses into clouds (liquid).

Deposition (Gas Solid)


Deposition occurs when a gas transforms directly into a solid, without going through
the liquid phase. Water vapor becomes ice or frost when the air touching a solid,
such as a blade of grass, is cooler than the rest of the air.

During the phase changes in matter, heat is either released or absorbed by the
system to the environment. Any change that involves the absorption of heat by the
system from the surroundings is called endothermic. Examples include evaporation,
sublimation and melting. Exothermic change on the other hand involves the
released of heat by the system into the environment. Examples are condensation,
deposition (solidification) and sublimation.

Note: Any process that proceeds endothermic in one direction will always be
exothermic in the opposite direction.

Properties of Matter

Various kinds of matter are distinguished from each other by their properties. A
property is a distinguishing characteristic of a substance that is used in its
identification and description. Properties of matter are of two general types:

Physical Property Chemical Property


• observable characteristics of a • characteristics of a substance
substance that describes the way it
• includes: color, physical state, undergoes or resists change to
melting and boiling points and form new substance
hardness • includes: reactivity, flammability,
• the paper‘s color is white corrosiveness and acidity of a
• water‘s boiling point is 100⁰C substance
• gold doesn‘t rust as much as iron
• Gasoline is flammable

The Physical Property of matter is further categorized in two: Extensive and Intensive
Properties: Below is a comparison of the two:

~5~
Source:https://www.google.com/url?sa=i&url=http%3A% Source:https://www.google.com/url?sa=i&url=https%3A%2F%2Fstu
2F%2Fwww.examhill.com%2Fbasic-concept dylib.net%2Fdoc%2F9918475%2Fextensive-vs.-intensive properties

Changes in Matter

Changes in matter are common and familiar occurrences. Changes take place
when food is digested, paper is burned, and a pencil is sharpened. Like properties
of matter, changes in matter are also classified into two categories: physical and
chemical.

A physical change is a process in which a substance changes its physical


appearance but not its chemical composition. A new substance is never formed as
a result of a physical change. Examples include melting of ice, chopping of wood,
breaking a glass, etc.

Chemical change is the process in which a


substance undergoes a change in
composition. It always involves conversion
of materials into one or more new
substances, each of which has properties
and composition distinctly different from
those of the original materials. Rusting of iron
and ripening of fruit are few examples.

Source: http://image.slidesharecdn.com/chemicalreactions
091215034010-phpapp01/95/chemical-reactions-4-
728.jpg?cb=1260850896

Let us see if you understand the concept of Chemical and Physical Changes. Work
on these examples below. Put a CIRCLE on the pictures that manifest physical
change and a BOX on pictures illustrating a chemical change. Enjoy!

~6~
Source:https://www.google.com/url?sa=i&url=https%3A%2F%2Fwww.thinglink.com

Classifications of Matter

Courtesy:Christy Johannesson www.nisd.not/communicationarts/pages/chem

In addition to its classification by physical state, matter can also be classified in


terms of its chemical composition as pure substance or as mixture. A pure
substance is a single kind of matter that cannot be separated into other kinds of
matter by any physical means. All samples of a pure substance contain only that
particular substance and nothing else. Also, pure substance always has definite
and constant composition that dictates the properties of a particular substance
that do not vary under a given set of condition. Pure water for example, consists

~7~
only of water and nothing else. No matter what the source maybe, pure water
always boils at 100⁰ C and freezes at 0 ⁰C at normal condition.

Pure substances can be further categorized to Element and Compound. An


element cannot be broken down into simpler pure substances by any chemical
means such as chemical reaction, an electric current or hit. The metals gold and
silver, and oxygen gas are all elements.

Compound, in contrast, is a pure


substance that can be broken down
or decomposed into two or more
simpler pure substances by chemical
means. Water is a compound, which
by means of electric current, can be
broken down into the gases
hydrogen and oxygen, both of
which are elements. The ultimate
breakdown products for any
compound are elements. Elements
are chemically combined to form a
compound; as a result the resulting

compound‘s properties are always Source: https://image.slidesharecdn.com/elements-compounds-mixtures-


different from that of the elements‘
1222945537104981-9/95/elements-compounds-mixtures-68-728.jpg

properties. Water which is liquid is the chemical combination of two gases – oxygen
and hydrogen.

A mixture is a physical combination of two or more pure substances in which each


pure substance retains its own chemical identity. Components of a mixture retain
their identity because they are physically mixed rather than chemically combined,
and for that, these components can also be separated through physical means.
Consider a mixture of salt and pepper. Mixing these two substances neither change
the salt nor the pepper in any way, and these two substances easily distinguishable
from each other. You can also separate the salt crystals from the pepper bits
through physical means like straining for example.

Variable composition is another property of a mixture. Numerous different salt-


pepper mixtures can be made from more salt – less pepper or more pepper – less
salt mixtures.

Mixtures are also sub-classified into heterogeneous or homogeneous, which is


based on visual recognition of the mixture‘s components. Heterogeneous mixture
contains visibly different phases (parts), each of which has different properties.
Examples are pizza and mixture of oil and water, whose individual components can
be easily differentiated. Colloids and Suspension are sub-classification of
~8~
heterogeneous mixture. Mayonnaise and body lotions are examples of colloids;
while halo – halo and freshly made calamansi juice are suspensions (have
suspended particles or other substances that will settle on the bottom).

On the other note, homogeneous mixtures contain only one visibly distinct phase,
which has uniform properties all throughout. Components in a homogeneous
mixture cannot be visually distinguished. A salt – water mixture in which all salt has
dissolved has an appearance similar to that of pure water. You cannot distinguish
the salt particles from the water. Air has a uniform appearance and it is a mixture of
different gasses such as oxygen, hydrogen, carbon dioxide, nitrogen and many
more, of which each gas is indistinguishable from each other. These types of mixture
is called a solution, which also exist in three phases – solid solution (alloy), liquid
solution (salt water), and gaseous solution (air).

Methods of Physical Separation of Mixtures

Now that we know about the different kinds of mixtures, we are going to learn
about some ways of separating them. Mixtures are very useful. However, sometimes
we need to separate mixtures into their components. Since mixtures are formed by
the physical combining of substances, they can also be physically separated. The
most appropriate technique for separating a mixture depends on the type of
mixture and the physical properties of the components. Below are some methods
of separating components of mixture:

Separation Technique Example

Decanting involves separating a liquid


(supernatant) from a solid (sediment) by
gently pouring off the liquid.

Sieving involves separating a mixture based


on different sizes of components, where sort stones or pebbles
smaller fragments pass through holes in the from sand
sieve but large fragments do not.
Muddy water is
Filtration is a special form of sieving where filter
poured through a funnel
paper is used to trap very fine solid particles
lined with filter paper to
(residue) from the rest of a liquid or gas
remove the small sand
mixture (filtrate).
and clay particles.
Sea
water
is
Evaporation is used for recovering dissolved
substances from solutions by evaporating the
solvent and crystallising the solute.

allowed to stand in shallow pans,


~9~
the water gets heated by sunlight
and slowly turns into water vapour,
through evaporation, leaving solid
salt behind
Distillation is the process of separating the
components or substances from a liquid
mixture by using selective boiling and
condensation.

Magnetism can be used for separating


magnetic materials from non-magnetic
materials.
Separating iron from grains of sand
Chromatography separates mixtures based on
Separating
their differing levels of attraction to two
contents of an ink
different substances, known as the mobile
phase and the stationary phase.

Laws of Matter

Source:https://www.google.com/url?sa=i&url=https%3A%2F%2Fwww.slideserv Source:https://www.google.com/url?sa=i&url=https%3A%2
e.com%2Ftaylor%2Fchapter-4-atoms-powerpoint- F%2Fwww.slideserve.com%2Ftaylor%2Fchapter-4-atoms-
pptpresentation&psig=AOvVaw0VUveK673k-gQkPY0gax2- powerpoint-ppt-presentation&psig /05/en-classification-of-
matter.png

Source:https://slideplayer.com/slide/6278
648/21/images/42/John+Dalton+proposed
+this+in+1803..jpgmatter.png

~ 10 ~
Application

Activity1. States of Matter

Using what you just learned from your readings and connecting it with your own
experiences, choose any of the following activities and describe the properties of
the states of matter and give examples through:

a. A song (w/ the melody)


b. A poem
c. An Illustration

Activity2. Classification of Matter

Look around your house, pick up items that interest you (kitchen ingredients, rock,
paper, jewelleries, etc.) then group them according to their classification. Arrange
them in such a way that they follow the hierarchy of matter classification. To do this,
prepare beforehand, colourful papers for you to write the classifications as your
labels. Take photos of your work and upload it in our online learning platforms (LENS
Moodle/ FB Messenger Group Chat). Have fun!

My Write – up
Based on what you did on the task above, complete the table below by providing
what is asked.

Item/Substance Classification Justification

Feedback

Directions: Multiple Choice: Read each question carefully, then choose and
encircle the letter of the best answer.
1. Which of the following is a property of both gases and liquids?
~ 11 ~
a. definite shape c. definite volume
b. indefinite shape d. indefinite volume

2. The process by which water is turned into ice is called ____.


a. Refrigeration c. Freezing
b. Icing d. Condensation

3. Which of the following is a compound?


a. Cemented wall c. 24 – karat gold
b. Dry ice d. Air

4. Which of the following is in a liquid form?


a. Water vapour c. Smoke
b. Clouds d. Steam

5. Which of the following is a correct description for a sample of ice cubes in


water?
a. one phase present, one substance present
b. one phase present, two substances present
c. two phases present, one substance present
d. two phases present, two substances present

Test II. Identify each of the following as a Physical Change, Chemical Change,
Physical Property, or Chemical Property.

6. You leave your bicycle out in the rain and it rusts.

7. Diamond is the hardest natural substance.

8. Melting of candle

9. Cleaning pennies with vinegar

10. Acid in tomato sauce can corrode aluminium foil.

Congratulations on finishing the lesson!

References:
~ 12 ~
Bettelheim, Frederick and March, Jerry, 2005. Introduction to General Organic and
Biochemistry. 5th ed. Singapore; Thomson Learning Asia.

Brown, T. L. et. al. 2003. Chemistry the Central Science. 9th ed. Singapore: Pearson
Education Asia Pte. Ltd.

Stoker, S.H. 2013. Exploring general, Organic and Biological Chemistry. 2nd Ed.
Philippines; Cengage Learning Asia Pte Ltd.

Phases of Matter - Glenn Research Center - NASA


www.grc.nasa.gov › www › airplane › state

Matter: Definition & the Five States of Matter | Live Science


www.livescience.com>46506-states-of-matter

Physical and Chemical Properties of Matter | Boundless...


courses.lumenlearning.com>boundless-chemistry>chapter>physical

3.6: Changes in Matter – Physical and Chemical Changes


chem.libretexts.org

8 Hands – on Experiments to Teach Kids About Chemical


https://owlcation.com/stem/hands-on-experiments-to-learn-about-chemistry

1.2 Phases and Classification of Matter


https://openstax.org/books/chemistry-2e/pages/1-2-phases-and-classification-of-
matter

http://www.umich.edu/~nppcpub/resources/compendia/CHEMpdfs/CHEMII.pdf

https://owlcation.com/stem/hands-on-experiments-to-learn-about-chemistry

https://www.britannica.com/science/chemistry

Helmenstine, Anne Marie, Ph.D. "What Is the Importance of Chemistry?" ThoughtCo,


Feb. 11, 2020, thoughtco.com/what-is-the-importance-of-chemistry-604143.

~ 13 ~
LESSON 2: Atoms, Ions and Molecules

Introduction

One of the greatest discoveries in modern science that has unraveled the secrets of
matter is the discovery of the atom. Understanding the nature of the atom has
catalyzed breakthroughs in chemical technology which is a key to human
development. Moreover, unlocking the mysteries of the atom is vital in realizing
many of the innovations in the other allied sciences such as Biology, Physics,
Astronomy, Medicine and the like.
An atom is the basic unit of an element that can enter into chemical combination.
It is too small that the human eye can not see but with the use of modern
technology like the electron microscope, this can be made possible.
The atoms of the elements in the Periodic Table are neutral because they have
equal number of protons and neutrons. But somehow, if these atoms start to lose or
gain electrons, they become ions. This process undertaken by atoms is very
indispensable in the formation of compounds and molecules.

Activating Prior Learning

Observe three different kinds of leaves, flowers and fruits found in your backyard.
Describe their color, odor and taste and write your observation on the space
provided.
_________________________________________________________________________________
_________________________________________________________________________________
_________________________________________________________________________________
_________________________________________________________________________________

What could explain the difference in the color of the leaves, odor of the flowers
and taste of the fruit of the plant?
_________________________________________________________________________________
_________________________________________________________________________________
_________________________________________________________________________________
_________________________________________________________________________________

ATOMIC THEORY

At the end of the lesson, you should be able to:


 trace the historical development of the atom;
 describe the structure of the atom; and
 discuss the importance of the different components of the atom.

~ 14 ~
Everything that we know about the atom today is a collective contribution of the
different scientists who based their hypothesis of the atom to the early ideas of the
ancient Greeks.

 Democritus
In the 5th century B.C. Democritus
believed that matter consists of very
small particles which he called
atomos, a Greek word for uncuttable
or indivisible. He further posited that
these atoms are solid and different
atoms vary in size and shape. The
difference in the size and shape of
atoms explains the differing properties
of matter.

 John Dalton
In 1808, an English scientist and
school teacher, John Dalton has
formulated a precise definition of the
indivisible building blocks of matter
that we call atoms. Dalton‘s work
marked the beginning of the modern
era of chemistry. The hypotheses
about the nature of matter on which
Dalton‘s atomic theory is based can
be summarized as follows:

1. Elements are composed of extremely small particles called atoms.


Note: This was rejected by modern view of the atom when quarks were
discovered in 1964 by the physicists Murray Gell-Mann and George Zweig.
Today, quarks are regarded as the building blocks of matter.

2. All atoms of a given element are identical, having the same size, mass, and
chemical properties. The atoms of one element are different from the atoms of
all other elements.
Note: This was rejected by modern view of the atom because of the concept of
isotopes. Isotopes are substances that have the same number of protons but
differ in their number of neutrons. The variability in the number of neutrons in
the atom has caused its mass number and properties to differ.

~ 15 ~
3. Compounds are composed of atoms of more than one element. In any
compound, the ratio of the numbers of atoms of any two of the elements present
is either an integer or a simple fraction.
Note: This conforms to the Law of Definite Composition or Proportion formulated
Joseph Proust

4. A chemical reaction involves only the separation, combination, or


rearrangement of atoms; it does not result in their creation or destruction.
Note: This conforms to the Law of Conservation of Mass formulated by Antoine
Laurent Lavoisier.

 Joseph John Thomson

The plum pudding model of the atom


was proposed by J.J. Thomson in 1904.
He described an atom as a positive
sphere to which sea of electrons are
loosely embedded on its surface. He
suggested further that the number of
negative charged particles equal the
number of positive charged particles in
the atom making it to be neutral. In
addition to this, J.J. Thomson has
discovered electrons in 1897 using
cathode ray tube experiment. The
cathode ray tube is the forerunner of our
television tubes.

A diagram of J.J. Thomson's cathode ray tube. The ray originates at the cathode and
passes through a slit in the anode. The cathode ray is deflected away from the
negatively-charged electric plate, and towards the positively-charged electric plate.
The amount by which the ray was deflected by a magnetic field helped Thomson
determine the mass-to-charge ratio of the particles.

 Ernest Rutherford

~ 16 ~
Rutherford‘s model of the atom is also
known as the nuclear model of the atom.
This was proposed by New Zealand – born
physicist Ernest Rutherford in 1911. The
model described the atom is almost an
empty space composed of tiny, dense,
positively charged core called a nucleus, in
which nearly all the mass is concentrated,
around which the light,
negative constituents, called electrons,
circulate at some distance, much
like planets revolving around the Sun.

The following data obtained in his Thin Gold Foil experiment in 1899 were the basis in
describing an atom in his model:
1. Most of the alpha particles travelled straight through the thin gold foil. This
indicates that the atom is almost an empty space.
2. Few of the alpha particles returned back at the same side where they come
from. This indicates that the alpha particles hit a dense part of the atom at
the center known as the nucleus, the part of the atom where the mass is
concentrated.
3. Some of the alpha particles were deflected at an angle. The deflection of
the alpha particles is an indication of the positive charge of the nucleus of
the atom since alpha particles are positively charged. This conforms to the
―Law of Charges‖ by Charles Coulomb which states that ―like charges repel
and unlike charges attract‖

 Niels Bohr

Bohr model of the atom is also known as


―Planetary Model of the Atom‖ and was
proposed in 1913 by Danish physicist Niels
Bohr. In his model, he described an atom
as consisted of a positive nucleus
surrounded by electrons that are
revolving around it in circular quantized
~ 17 ~
orbit. The electrons are held in their orbit by centripetal force.

However, these electrons can jump into a higher energy levels when they absorbed
heat or through influence of electric current (excited state) and emit
electromagnetic radiation (photon) with corresponding wavelength when they
release the absorbed heat (ground state).

 Erwin Scrodinger and Werner Heisenberg


The electron cloud model is the current
accepted model of an atom. This was
developed by Erwin Scrodinger and
Werner Heisenberg in 1926. They
described the atom as consisted of a
dense nucleus surrounded by a cloud
of fast spinning electrons at various
levels in orbitals. The electron cloud or
orbital is the region in the atom where
one
could get 90% chance of finding the
electrons.
However, the model cannot determine
the exact location and momentum of
electrons at the same time (Heisenberg Uncertainty Principle). To solve this
dilemma, Schrodinger proposed the following four (4) sets of quantum numbers:

a. Principal Quantum Number – its symbol is n and has integral values 1, 2, 3 and
so forth. It is related to the average distance of the electron from the nucleus
in a particular orbital.
Example: In the outer electron configuration of Oxygen, 2p4, what is the n
value? The answer is n = 2. Remember that in electron configuration, it follows
the symbol nlx.

b. Azimuthal or Angular Quantum number – its symbol is l and its values depend
on the orbital where the electron is located. The l value for s orbital is always
0, for p orbital, it is always 1, d orbital is always 2 and f orbital is always 3. This
quantum number defines the shape of the sublevel of the atom to which the
electron is found.

~ 18 ~
Example: What is the l value in the above example? The answer is l = 1
because the electron is in the p sublevel.

c. Magnetic Quantum number – its symbol is ml (read as m sub l). It describes the
probable orientation of orbital in space. Within a subshell, the value of m l
depends on the value of the angular momentum quantum number, l. For a
certain value of l, there are (2l + 1) integral values of ml as follows:
sublevel L value ml = (2l + 1) ml possible values
s 0 1 0
p 1 3 -1, 0, +1
d 2 5 -2, -1, 0, +1, +2
f 3 7 -3, -2, -1, 0, +1, +2, +3

Example: Give the ml value of the electrons in the outer configuration of the
atom Oxygen – 2p4.
Solution: Since the electrons are in the p sublevels, there are 3 possible values,
it can be -1, 0 or +1.
Note: For ml of specific electron in the orbital, use orbital box diagram to
determine the ml value.

Example: What is the ml value of the last entering electron of Oxygen atom?
(This means electron # 4 in 2p4 configuration)
Solution: Since the electron is in 2p orbital, the third column of the table
above tells that there are 3 orbital present in the sublevel, therefore, in the
orbital box diagram, distribute the 4 electrons in the 2p 4 configuration in this
manner:

So the answer is ml = -1

d. Spin Quantum Number – describes the angular momentum of electron and


has only two possible values, +1/2 and -1/2. You should remember that both
values are accepted for every electron in the atom. Its symbol is m s (read as
m sub s).
Example: What is the ms value of the last entering electron in Oxygen atom
with an outer configuration of 2p4? The answer is either +1/2 or -1/2

Activity 1

1. Construct a miniature of the atom using indigenous materials.

~ 19 ~
2. Determine the four sets of quantum numbers of the electrons in 3d7.
3. Determine the 4 sets of quantum numbers in the last entering electron of 4p 5.

ATOMIC STRUCTURE
Atoms are tiny particles of matter (about 10 -10 m in diameter or approximately 1 pm
in diameter). They are the basic unit of matter and the defining structure of an
element. Atoms are composed of the following particles:

a. Protons are positively charged particle of the atom. They are found inside
the nucleus and contributory particles for the mass of the atom. These
particles are discovered by Ernest Rutherford and give atoms their identity.

b. Neutrons are uncharged particles that are found in the nucleus of the atom
and were discovered by James Chadwick. These particles contribute to the
mass of the atom and stabilize the nucleus by effectively moderating the
repulsive force among the protons in it.
Protons and neutrons are collectively called nucleons because they are
particles of the nucleus of the atom. Protons bind with neutron in the nucleus
using strong nuclear force. The nucleus is positively charged and the region
in the atom where the mass is concentrated.
c. Electrons are negatively charged particles which are discovered by J.J.
Thomson and are found outside the nucleus of the atom. These particles are
not contributory to the mass of the atom but are essential participants during
chemical reaction forming various compounds.

Properties of Sub – atomic Particles


Property Electron Proton Neutron
Symbol e- p+ n
Mass (amu) 0.000549 1.00728 1.00867
Mass (g) 9.1094 x 10 -28 1.6726 x 10 -24 1.6749 x 10 -24
Mass (kg) 9.109 × 10-31 1.673 × 10-27 1.675 × 10-27
~ 20 ~
Electric Charge -1 +1 0
1 amu = 1.6605 x 10 -24 g

Activity 2
1. Compose a jingle that features an atom, its components and their function.
2. Construct a timeline showing the development of the atom.

ELEMENTARY PARTICLES
Electrons and quarks contain no discernible structure and cannot be reduced or
separated into smaller components. It is therefore reasonable to call these particles
elementary particles. The use of modern high – technology equipment today has
helped us understand better the atom and its fundamental particles. Today,
modern view of the atom has proven that it is comprised of many other elementary
particles aside from what we have known before.

The foundation of this innovation in atomic structure is attributed to the idea of Paul
Dirac who predicted in 1928 that ―all particles should have their opposites called
the anti – particles‖. The first of this was discovered in 1932 by Carl Anderson, which
he called positron – the anti – particle of electron. When an electron and positron
come into contact, they mutually annihilate each other producing a flood of
energy in accordance with Einstein's famous equation, E = mc2.

 Leptons
Leptons are never found in the nucleus of atoms. They are not subject to the
Strong Nuclear Force which keeps the nucleus from flying apart. There are six
(6) types of leptons which include the electron, muon, tau and the neutrino
having its three types known as electron neutrino, muon neutrino and tau
neutrino. The electron is considered the simplest of the leptons. Neutrino on
the other hand was postulated in 1934 by Enrico Fermi to explain certain
aspect of radioactive decay.

 Baryons
The two most common baryons are the proton and neutron. Both are found
in the nuclei of atoms, being kept there by the Strong Nuclear Force that
binds them together. Baryons are made up of even more elementary
particles called quarks. A baryon particle is composed of quark triplet. An
example is proton which is composed of 2 u (up) quarks and a d (down)
quark. Neutron on the other hand is composed of 2 d quark and a u quark.
Quarks are very unususal because they have fractional electric charges. The
are 6 types or flavors of quarks which include the up, down, strangeness,
bottom (or beauty) and top (or truth).

The Six Types of Quarks

Name of Quark Symbol Charge Mass (MeV)

Up u +(2/3) 2–8

~ 21 ~
Down d -(1/3) 5 – 15

Strangeness s -(1/3) 100 – 300

Charm c +(2/3) 1,000 – 1,600

Bottom (or Beauty) b -(1/3) 4,100 – 4,500

Top (or Truth) t +(2/3) 180,000

The Baryon Particles and their Charges

Baryon Particle Quark Triplet Charge

p (proton) uud +(2/3)+(2/3)-(1/3) = +1

n (neutron) udd +(2/3)-(1/3)-(1/3) = 0

- ddd -(1/3)-(1/3)-(1/3) = -1

0 uds +(2/3)-(1/3)-(1/3) = 0

+ uus +(2/3)+(2/3)-(1/3) = +1

- sss -(1/3)-(1/3)-(1/3) = -1

C1++ cuu +(2/3)+(2/3)+(2/3) = +2

 Mesons
Mesons are particles only discovered when the forces binding nucleons
together were investigated. In a nucleus, the protons and neutrons are not
really separate entities, each with its own distinct identity. They change into
each other by rapidly passing particles called pions ( ) between themselves.
Pions are the most common of the mesons. Mesons are composed of a quark
/ anti-quark pair. The positive pion ( +) is made from a u quark and and a (d)

~ 22 ~
anti quark. The negative pion ( -) is made from a d quark and a (u) anti
quark.
Based on spin of the particle, the elementary particles can be categorized
into Fermions and Bosons. Fermions have spin that have half – integer values
like 1/2, 1/3, 3/2 etc. This characteristic spin of Fermion proves that they obey
Pauli‘s Exclusion Principle. Leptons and baryons belong to this category.
Bosons on the other hand, have integer spin (0, 1, 2 etc.) Bosons do not obey
the Pauli Exclusion Principle. The best known Boson is the massless photon, a
quantum of light.

Activity 3

Construct a concept map that shows the different categories of elementary


particles of the atom.

ATOMIC NUMBER AND ATOMIC MASSES

The atomic number of an element is the number of protons found in the nucleus
of its atom. Its symbol is Z. For a neutral atom, the number of proton is always
equal to the number of electron in the atom.

Example: The atomic number of Oxygen is 8. How many protons and electrons
are there in its atom?

Answer: There are 8 protons in its nucleus and 8 electrons circling around it. The
number of protons equals the number of electrons since the atom is neutral.

Atomic Mass

The atomic mass of an element can be determined by adding the number of


protons and neutrons in the nucleus of its atom. Its symbol is A.

Example 1: The atomic number of Oxygen is 8 and there are 8 neutron in its
nucleus. What is the atomic mass of Oxygen atom?

Answer: Adding the atomic number and number of neutron, A = 8 + 8 = 16. So


atomic mass is 16. This value coincides with the atomic mass of Oxygen as seen
on the periodic table of elements when rounded off to whole number.

Example 2. The nuclear symbol for an atom of Tungsten is , how many


protons, electrons and neutrons are there in the atom?

Answer: The nuclear symbol of an atom is represented as , where A is the


Atomic mass and Z is the atomic number. Therefore, p = 74, e = 74 and n= A – z ,
-

n = 184 – 74 = 110

~ 23 ~
You should remember that atoms of a given element do not have the same mass.
Most elements have two or more isotopes – atoms that have the same atomic
number but different mass numbers or same number of protons but differ in the
number of neutrons. For example: The three isotopes of Hydrogen are: a.) protium,
, has 1 proton and 0 neutron; b.) deuterium , has 1 proton and 1 neutron and
c.) tritium , has 1 proton and 2 neutrons.

Look at the atomic mass of the elements on the periodic table, the value that you
see is the average atomic mass of the element. Example, on the periodic table, the
atomic mass of Carbon is 12.011. This value is the average mass of the naturally
occurring mixture of isotopes of Carbon. How to arrive at the average mass of
Carbon?

Solution: the natural abundances of carbon-12 and carbon-13, the isotopes of


Carbon are 98.90 % and 1.10 %, respectively. The atomic mass of carbon – 12 is
12.00000 amu and carbon – 13 has been determined to be 13.00335 amu.
Therefore:

average atomic mass of natural carbon = (0.9890)(12.00000 amu) +


(0.0110)(13.00335 amu) = 12.011 amu

Activity 4
Solve the following problems:
1. Copper, a metal known since ancient times, is used in electrical cables and
pennies, among other things. The atomic masses of its two stable isotopes,
(69.09 %) and (30.91 %), are 62.93 amu and 64.9278 amu,
respectively. Calculate the average atomic mass of copper. The relative
abundances are given in parentheses.

2. The atomic masses of the two stable isotopes of boron, (19.78 percent)
and (80.22 percent), are 10.0129 amu and 11.0093 amu, respectively.
Calculate the average atomic mass of boron.

Look and analyzed the nuclear symbols of the following chemical substances:

These substances are classified as isobar. They have the same atomic mass
(number of nucleons) but differ in their atomic number (number of protons).

IONS
Atoms can become ion if it loses or gains an electron. Ions are substances that
carry a charge. The charge is due to the unequal number of electrons and protons
in the atom. An ion that carries a positive charge is called a cation while an anion

~ 24 ~
carries a negative charge. The positive charge of an atom indicates that it has
more proton than electron and negative charge indicates that the atom has more
electrons than protons

Metals are regarded as electron donors, therefore, they tend to become cations.
Examples are Li+ (read as lithium ion), Na+ (read as Sodium ion), K+, Be2+ , Mg2+ , Ca2+
etc. Non-metals on the other hand are electron acceptors, therefore, they tend to
become anions. Examples include C4- (read as carbide ion), N3- (read as nitride
ion), O2- (read as oxide ion) etc.

The formation of ions by substances is a process undertaken to achieve stability.


Stability is attained when the outermost energy level of the atom is completely
filled.

What happens to the number of protons, electrons and neutrons in an ion?

Example 1: How many protons, electrons and neutrons are present in an atom with
a nuclear symbol 3+ ?

Answer: p = 13, e- = 10, n = 14 Note: Al carries a 3+ charge. This indicates that Al has
lose 3 e-. Therefore the difference between its number of protons and electron
should be 3.

Example 2: What is the number of protons, electrons and neutron in an atom with a
nuclear symbol 2-?

Answer: p = 16, e- = 18, n = 16 Note: S carries 2- charge. This indicates that S has
gained 2 electrons. Therefore, the difference between its number of protons and
electron should be 2.

Activity 5:

Complete the following table:

Element Atomic Atomic # of # of # of Nuclear


Number (Z) Mass (A) protons electrons neutrons Symbol

Mg 12 12 12

Cl 17 18 18

Ag 47 108 61

I -

Pt

 MOLECULES

A molecule is an aggregate of at least two atoms in a definite arrangement held


together by chemical forces (also called chemical bonds). A molecule may
contain atoms of the same element or atoms of two or more elements joined in a
fixed ratio, in accordance with the law of definite proportions. Thus, a molecule is

~ 25 ~
not necessarily a compound, which, by definition, is made up of two or more
elements. Hydrogen gas, for example, is a pure element, but it consists of
molecules made up of two H atoms each. Water, on the other hand, is a
molecular compound that contains hydrogen and oxygen in a ratio of two H
atoms and one O atom. Like atoms, molecules are electrically neutral. The
hydrogen molecule, symbolized as H2, is called a diatomic molecule because it
contains only two atoms. Other elements that normally exist as diatomic
molecules are nitrogen (N2) and oxygen (O2), as well as the Group 7A elements—
fluorine (F2), chlorine (Cl 2), bromine (Br2), and iodine (I2). Of course, a diatomic
molecule can contain atoms of different elements. Examples are hydrogen
chloride (HCl) and carbon monoxide (CO). The vast majority of molecules
contain more than two atoms. They can be atoms of the same element, as in
ozone (O3), which is made up of three atoms of oxygen, or they can be
combinations of two or more different elements. Molecules containing more than
two atoms are called polyatomic molecules. Like ozone, water (H 2O) and
ammonia (NH3) are polyatomic molecules.

Activity 6

Classify the following substances by putting a checking mark if it is a molecule


and a cross mark if it is not.
Substances Classification
MgCl2
Al(OH)3
C2H5OH
Na2C
H2SO4
SO2
SF6
HCOOH
P2O5
FeCl 3

Assessment

~ 26 ~
I. Multiple Choice: Choose the best answer from the given choices. Write the letter
of our choice on the space provided.
_____1. Which of the following shows the CORRECT order of Atomic Theory Timeline?
a. Bohr, Democritus, Thomson, Rutherford, Dalton
b. Rutherford, Bohr, Dalton, Democritus, Thomson
c. Dalton, Thomson, Rutherford, Bohr, Democritus
d. Democritus, Dalton, Thomson, Rutherford, Bohr

____2. Atoms of 16O, 17O, and 18O have the same number of
a. protons, but a different number of electrons
b.protons, but a different number of neutrons
c. electrons, but a different number of protons
d. neutrons, but a different number of protons

____3. A sample of element X contains 90% X-35 atoms, 8.0% X-37 atoms, and 2.0%
X-38 atoms. The average
atomic mass will be closest to which value?
a. 35 b. 36 c. 37 d. 38

_____4. Which two atoms are isotopes of one another?

Atom # of proton # of neutron # of electron


A 8 10 8
B 10 9 10
C 9 9 9
D 8 11 8

a. A & B b. A & C c. B & C d. A & D

____5. The atomic theories of Dalton, Thomson, Rutherford, and Bohr all support
which of the following statements?
a. atoms are mostly composed of empty space
b. electrons orbit the nucleus of an atom at distinct energy
levels
c. all matter is composed of tiny, discrete particles called
atoms
d. atoms are composed of positively and negatively charged
particles.

____6. Deo counted the number of sub – atomic particles of


element X as follows:
Proton = 42 Electron = 44 Neutron = 48
Which of the following is a correct nuclear representation of
element X?
a. b. c. 2+ d. 2-

____7. Teacher Elisha calls on students to characterize protons of an atom. Which of


the following would be an acceptable answer?
a. Protons are involved during chemical reaction forming
various products.

~ 27 ~
b. The highly reactive electrons are stabilized by the proton
particles.
c. Every atom is identified by the number of protons inside its
nucleus.
d. The mass of the atom is given by the total mass of protons in the nucleus.
____8. A hypothetical element named Pedronium was found to
have 172 protons, 172 electrons and 205 neutrons. What
would be the atomic number of Pedronium?
a. 33 b. 172 c. 205 d. 377

____9. Which ot the following atomic particles are similar in number


in the following species?
Cl-, Ar, and K+

a. number of protons c. number of electrons


b. number of neutrons d. number of isotope

_____10. Which of the following shows the quark triplet composition


of a proton particle?
a. udd b. uud c. usd d. csu

_____11. Rutherford carried out experiments in which a beam of


alpha particles was directed at a thin piece of metal foil.
From these experiments he concluded that:
a. electrons are massive particles.
b. the positively charged parts of atoms are moving about
with a velocity approaching the speed of light
c. the positively charged parts of atoms are extremely small
and extremely heavy particles.
d. the diameter of an electron is approximately equal to
that of the nucleus.

_____12. Which of the responses contain all the statements that are
consistent with the Bohr‘s theory of the atom?
1. An electron can remain in a particular orbit as long as it continually absorbs
radiation of a definite frequency.
2. The lowest energy orbits are those closest to the nucleus.
3. An electron can jump from the K shell (n = 1 major energy
level) to the M shell (n = 3 major energy level) by emitting
radiation of a definite frequency.
a. 1 only b. 2 only c. 3 only d. 2,3

____13. Electrons are either lose or gained by atoms to form ions. What is the driving
force for this mechanism of the atom?
a. Reactivity b. Stability c. Identity d. Volatility

____14. Which of the following sets of quantum numbers is not


allowed?
a. n = 3, l = 1, ml = -1, ms = + ½

~ 28 ~
b. n = 3, l = 0, ml = 0, ms = + ½
c. n = 2, l = 1, ml = 0, ms = + ½
d. n = 2, l = 2, ml = -1, ms = - ½

____15. Which of the following substances is NOT a molecule?


a. ZnBr2 b. PH3 c. CH4 d. CO2

____16. Niels Bohr concluded that atoms are similar with the solar system in its
orientation. Electrons surround the nucleus of the atom and do not fall away
from the nucleus. What possible explanation supports this statement?
a. Electrons are held on their position by their attraction with the nucleus.
b. Electrons are pulled closer to the nucleus by the neutron of the atom.
c. Protons and electrons on the orbit of the atom established strong attractive
force.
d. The electrons are being fixed on their position by the different quarks in the
nucleus.

____17. Rutherford observed that MOST of the alpha particles went through the thin
gold foil without being deflected. What is indicated by this observation of
Rutherford to atoms?
a. The atom is completely a gaseous object consisted of extremely small
particles.
b. The atom is so big and all its small particles are concentrated at the center.
c. The protons, electrons and neutrons are randomly distributed occupying the
entire space of the atom.
d. The volume of an atom is almost an empty space.

____18. The atomic masses of and are 6.0151 amu and 7.0160 amu,
respectively. If the average atomic mass of Li is 6.941 amu, what are the natural
abundances of these two isotopes?
a. = 3.25 %, 96.75 % c. = 7.50 %, 92.50 %
b. = 5.50 %, 94.50 % d. = 9.25 %, 90.75 %

____19. Which of the following ions has gained 3 electrons?


a. Sr2+ b. S2- c. Ga3+ d. P3-

____20. What sub – atomic particle stabilizes the nucleus of the atom by effectively
moderating the repulsive force among the protons in it?
a. Proton b. electron c, neutron d. positron

References:
Chang, R. Chemistry.(2010). 5th ed. Mc Graw Hill Book Co.. N.Y.

Manahan, Stanley E. (1993) Fundamentals of Chemistry. London: Lewis.

Masterton and Hurley. (2004). Chemistry: Principles and Reactions. 5th ed. Thomson
Learning, Inc.

~ 29 ~
Purdue University. (22002). Special Edition of General Chemistry. McGraw-Hill Primis
Custom Publishing.
http://www.cse.salford.ac.uk/physics/gsmcdonald/pp/PPLATOResources/h-
flap/p8_1t.pdf
https://www.unf.edu/~michael.lufaso/chem2045/Chapter2.pdf
https://www.khanacademy.org/science/chemistry/electronic-structure-of-
atoms/history-of-atomic-structure/a/daltons-atomic-theory-version-2
https://www.britannica.com/science/Rutherford-model
https://www.toppr.com/guides/chemistry/structure-of-atom/rutherfords-model-of-
an-atom/
https://www.google.com/search?q=thin+gold+foil+experiment&rlz=1C1RLNS_enPH
872PH872&sxsrf=ALeKk000Khk1wlnzq1QpHnMYB_RWxwlQfA:1597542915463&source=
lnms&tbm=isch&sa=X&ved=2ahUKEwjghfWPz57rAhUMEqYKHUJVDE0Q_AUoAXoECB
QQAw&biw=1280&bih=610#imgrc=8Dppj5aoUHteZM
https://www.khanacademy.org/science/chemistry/electronic-structure-of-
atoms/history-of-atomic-structure/a/discovery-of-the-electron-and-nucleus
https://www.youtube.com/watch?v=B4cGRjvEpp0
https://courses.lumenlearning.com/boundless-chemistry/chapter/the-structure-of-
the-atom/

~ 30 ~
Lesson 3: THE PERIODIC TABLE AND ELECTRONIC STRUCTURE OF ATOM

Introduction

The periodic table of elements is tabular displays of the 118 known elements, the
first 92 of these elements are naturally occurring and the remaining elements are
synthetically produced in the laboratory. It has undergone series of development
through the resilient efforts of various chemists. Nonetheless, the present periodic
table of elements that you use today is based on the work of Dmitri Mendeleev. The
elements in the periodic table are arranged according to increasing atomic
number which become the basis of predicting the chemical properties of the
elements. The table shows the chemical symbol, atomic number, atomic mass,
electron configuration and electronegativity value of the element.
Looking at the periodic table will give you information on the electronic structure
and some of its properties like metallic character, atomic and ionic size, ionization
energy, electronegativity, electron affinity and chemical reactivity.

Learning Outcomes:

At the end of the lesson, you should be able to:


 trace the development of the periodic table;
 explain the trends in properties of elements based on its position on the
periodic table;
 illustrate the application of Aufbau, Hund‘s Rule and Pauli‘s Exclusion
principle in electron configuration; and
 predict the period and group based on the valence electron obtained from
electron configuration of the element.

Activating Prior Learning

Study the information written on the mail envelope and answer the following
question:

From: PEDRO P. SUYU


#35 Ramirez St, Parabba, Penablanca Cagayan
Cagayan Valley, Philippines
3502

To: Zarina Kate Laggui


1234 - E Main St. STE B6
San Francisco CA
94104-1207

What is the significance of the name and address of the recipient on the mail
envelope?
_________________________________________________________________________________
_________________________________________________________________________________
_________________________________________________________________________________
_____________________

~ 31 ~
Presentation of the Content

 HISTORICAL DEVELOPMENT OF THE PERIODIC TABLE


 1829 - German chemist Johannes Wolfgang Dobereiner noticed that many of
the known elements could be grouped in triads (a set of three elements that
have similar properties)—for example, chlorine, bromine, and iodine; or
copper, silver, and gold. However, subsequent attempts to expand his
concept on triad of elements were unsuccessful.

 1862 - French geologist Alexandre-Emile Béguyer de Chancourtois plotted the


atomic weights of elements on paper tape and wound them, spiral like,
around a cylinder. The design put similar elements onto corresponding points
above and below one another. He called his model the ―telluric helix‖ or
screw.

 1864 - English chemist John Newlands noticed that, if the elements were
arranged in order of atomic weight, there was a periodic similarity every 7
elements. He proposed his ‗law of octaves‘. Noble gases had yet to be
discovered, which is why Newland‘s table had a periodicity of 7 rather than 8.

 1868 - Lothar Meyer compiled a periodic table of 56 elements based on a


regular repeating pattern of physical properties such as molar volume. Once
again, the elements were arranged in order of increasing atomic weights.

 1869 - Russian chemist Dmitri Mendeleev produced a periodic table based


on atomic weights but arranged ‗periodically‘. Elements with similar

~ 32 ~
properties appeared under each other. Gaps were left for yet to be
discovered elements.

 1894 – William Ramsay discovered the noble gases and realized that they
represented a new group in the periodic table. The noble gases added
further proof to the accuracy of Mendeleev‘s table.

 1913 – Henry Moseley determined the atomic number of each of the known
elements. He realized that, if the elements were arranged in order of
increasing atomic number rather than atomic weight, they gave a better fit
within the ‗periodic table‘ which becomes the basis for the Periodic Law
which states that the properties of elements are periodic functions of their
atomic number.

 1928 – Amateur French scientist Charles Janet uses mathematical patterns to


investigate the electron configuration of elements. He groups elements into
blocks named after their atomic orbitals: s-block (sharp), p-block (principal),
d-block (diffuse) and f-block (fundamental).

 1944 - Glenn Seaborg proposed an ‗actinide hypothesis‘ and published his


version of the table in 1945. The lanthanide and actinide series form the two
rows under the periodic table of elements.

Activity 1
Search from the net other proposed arrangement of elements and select one to be
presented to the group. Discuss the salient features of your selection.

 Name and Symbols

Each element was given a name that is based on:


a. Property of the element - The Latin name for gold is aurum, meaning "shining
dawn." The Latin name for mercury is hydrargyrum, which means "liquid silver."
Cesium comes from the Latin word caesius, meaning "sky blue."
b. Name of a person - Curium - Marie Curie, Seaborgium – Seaborg and
Rutherfordium - Rutherford
c. Place of discovery – the four elements terbium, yttrium, erbium, and ytterbium
are named after the small town in Sweden known as Ytterby. Californium is
another example, produced in the Radiation Laboratory at the University of
California.

~ 33 ~
d. Celestial body – Plutonium is named after Pluto and Cerium named after
Ceres.
e. Mineral – Aluminum comes from the word alumen, Latin name for alum and
Beryllium comes from the word beryllos – the Greek word for beryl.
f. Color – Indium comes from the word iris – Latin word for rainbow and Iodine
from the word ioeides – Greek word for violet colored.
g. Mythology – Promethium from Prometheus in Greek Mythology and Thorium
from Thor in Scandinavian Mythology.

The symbol of the name of the element was obtained from:


a. the first letter of its name or of its Latin name
Example: H – Hydrogen, B – Boron, C – Carbon, O – Oxygen, K – Kalium and W
– Wolfram
b. First 2 letters of its name or Latin name
Example: Li – Lithium, Ca – Calcium, Al – Aluminum, Na – Kalium, Fe – Ferrum
c. First and 3rd letter of its name
Example: Cl – Chlorine, Mg – Magnesium, Zn – Zinc, Ag Argentum
The first letter of the symbol of the name of the element is ALWAYS written in
uppercase and the second letter is in lowercase.

Activity 2:
Give other examples of elements that were named based on the scheme
described above.

Activity 3:
Give examples of symbol of elements that follow the scheme describe above and
include examples of symbol of element obtained from 1st and 4rth letter, 1st and 5th
letter, 1st and 6th letter and 1st and 7th letter of its name.

 Group and Period


Elements in the periodic table are categorized in two ways. These are the groups
and periods of the periodic table.
A. Groups of Elements
Group or Family is the vertical column of elements in the periodic table. There are
18 columns that corresponds to 18 groups or families in the periodic table as
designated as Group 1 to 18 by the International Union of Pure and Applied
Chemistry (IUPAC). However, the traditional system makes use of Group A and B to
designate the group of elements. The s and p fillers or commonly known as main
group of elements or representative elements are group A elements while transition
metals belong to the B group of elements. The group of an element is given by the
number of valence electron of the atom in an element. Valence electrons are the
electrons found at the outermost energy level of the atom. Elements belonging to
the same group manifest similar chemical properties.

~ 34 ~
Element Symbol Atomic No. Electron No. of Group Number
No. (Z) of e- Configuration Valence e-
Traditional IUPAC
System System
Lithium Li 3 3 1s2 2s1 1 IA 1
Beryllium Be 4 4 1s2 2s2 2 IIA 2
Aluminum Al 13 13 1s2 2s2sp6 3 IIIA 13
3s23p1
Silicon Si 14 14 1s2 2s2sp6 4 IVA 14
3s23p2
Arsenic As 33 33 1s2 2s22p6 5 VA 15
3s23p6
4s 3d104p3
2

Selenium Se 34 34 1s2 2s22p6 6 VIA 16


3s23p6
4s 3d104p4
2

Iodine I 53 53 1s2 2s22p6 7 VIIA 17


3s23p6
4s 3d104p6
2

5s24d105p5
Xenon Xe 54 54 1s2 2s22p6 8 VIIIA or 0 18
3s23p6
4s 3d104p6
2

5s24d105p6
Scandium Sc 21 21 1s2 2s22p6 3 IIIB 3
3s23p6 4s23d1
Titanium Ti 22 22 1s2 2s22p6 4 IVB 4
3s23p6 4s23d2
Vanadium V 23 23 1s2 2s22p6 5 VB 5
3s23p6 4s23d3
Chromium Cr 24 24 1s2 2s22p6 6 VIB 6
3s23p6 4s23d4
Manganes Mn 25 25 1s2 2s22p6 7 VIIB 7
e 3s23p6 4s23d5
Iron Fe 26 26 1s2 2s22p6 8 VIIIB 8
3s23p6 4s23d6
Cobalt Co 27 27 1s2 2s22p6 9 VIIIB 9
3s23p6 4s23d7
Nickel Ni 28 28 1s2 2s22p6 10 VIIIB 10
3s23p6 4s23d8
Copper Cu 29 29 1s2 2s22p6 11 IB 11
3s23p6 4s23d9
Zinc Zn 30 30 1s2 2s22p6 12 IIB 12
3s23p6 4s23d10

~ 35 ~
Names of the different Main Group Elements
Group 1 or IA – Alkali Metals
Group 2 or IIA – Alkaline Earth Metals
Group13 or IIIA – Boron Family
Group 14 or IVA – Carbon Family
Group 15 or VA – Nitrogen Family
Group 16 or VIA – Chalcogen or Oxygen Family
Group 17 or VIIA – Halogen Family (salt formers)
Group 18 or VIIIA or 0 – Noble or Inert Gases (Unreactive)
Inert gases or noble gases are special group of elements in the periodic table
because these are the only elements that have fully filled outer energy level. As a
consequence, these elements are regarded as unreactive. However, when these
elements are subject to extreme heat and pressure, they become ionized and may
enter into chemical reaction forming compounds like XeO 3, XeO4, XeCl 2 etc.

Activity 4
Complete the following table:
Group of elements Description Application
1 or IA
2 or IIA
13 or IIIA
14 or IVA
15 or VA
16 or VIA
17 or VIIA
18 or VIIIA or 0
Inner transition
metals
Late transition
metals

B. Periods of the elements


The period or series is the horizontal row of elements in the periodic table. There are
7 periods included in the periodic table. The lanthanide series belongs to period 6
while the actinide series belongs to period 7. The period represents the main energy
level of the electrons in the atom. In an electron configuration, the coefficient gives
the period of the electron in the atom and gives information on its distance from
the nucleus.
For example in Oxygen atom where it has 8 electrons (Atomic number is 8), its
electron configuration is 1s2 2s22p4.
Interpretation: There are 2 main energy levels present in the atom. The inner energy
level (n=1 or K shell) has 2 electrons and the outer energy level (n=2 or L shell) has 6
electron from the s sublevel and p sublevel. The valence electrons of Oxygen atom
are occupying the 2nd energy level, therefore, its period is 2 and coincides with its
position on the periodic table.

~ 36 ~
Elements are also grouped into metals, non – metals and metalloids in the periodic
table. Almost 80 % of the elements in the periodic are metals, 15 % is non – metal
and the rest are metalloids.
a. Metals – hard, dense, shiny and durable which are good conductors of heat
and electricity. They are found left of the zig-zag line of the periodic table.
They readily lose electrons (electron donor) and form basic oxides when they
react with water. These elements manifest the following properties:
1. Luster – ability to reflect light on its surface.
2. Malleability – ability to be flattened into thin sheets.
3. Ductility – ability to be drawn into wires
4. Elasticity - refers to the rate at which a given metal sample is able to distort
its size and shape under a range of stress and strain forces and other
externally varying factors.
b. Non – metals – lightweight, brittle and poor conductor of heat and electricity
(insulator). They are found right of the zig-zag line of the periodic table. They
readily accept electrons (electron acceptor) and form acids when they
react with water.
c. Metalloids – elements lying along the zig-zag line. They have properties that
are intermediate between those of metals and nonmetals. Metalloids can
also be called semimetals. Examples are boron, silicon, germanium, arsenic,
antimony, tellurium, and polonium.
Activity 5
Research on the distribution of elements in the earth‘s crust.

Activity 6
Research on the elemental constituents of the earth‘s atmosphere.

Activity 7.
Research on the industrial application of metalloids.

 Trends in the Periodic Table

The position of elements in the periodic table explains the variation in the
properties manifested by the different elements. These properties include metallic
character, atomic and ionic size, ionization energy, electronegativity and
electron affinity.

~ 37 ~
A. Metallic Character
Trend: Metallic character increases from right to left and top to bottom of the
periodic table. The most metallic elements are found lower left of the periodic
table.
Example: Na is more metallic than Mg but K is more metallic than Na.
B. Atomic radius - is the distance from the center of the nucleus to the outermost
shell containing electrons.
Trend: Atomic radius increases from right to left and top to bottom of the
periodic table. The biggest atoms are found lower left of the periodic table.
Reason: Across a period, atomic size decreases because of increase
effective nuclear charge and down a group, atomic size increases because of
added energy level.
Effective nuclear charge or Zeff is the strength of attraction between the protons in
the nucleus and the electrons outside the nucleus. The greater the effective
nuclear charge, the smaller the atomic radius of the atom. The increase in effective
nuclear charge of elements across a period is explained by the decrease in the
shielding effect in the electrons as the number of electrons increases.
Shielding effect describes the shielding of electrons with each other from the pull of
the nucleus towards itself. Therefore, the higher the shielding effect, the weaker the
pull of the nucleus and the bigger the atomic size.
Example: Na is bigger in atomic size than Ca but K is bigger than Na.
Explanation: The valence electrons of Ca are poorly shielded by the inner electrons,
therefore effective nuclear charge of Ca is higher than Na causing its atom to shrink
and becomes smaller than Na. On the other hand, K is bigger that Na because K
belongs to period 4 and Na belongs to period 3. Therefore, K has 1 more energy
level than Na.
Ionic Size
Trend: Cation is always smaller that its neutral atom while anion is always
bigger than its neutral atom.
Example: Which is bigger, Na or Na +? O or O2-?
Na is bigger that Na+. When Na loses 1 electron, it achieves the configuration of Ne
which belongs to period 2 and has only 2 energy levels. Na belongs to period 3 and
has 3 main energy level. Therefore, Na has1 more energy level that Na+ .
On the other hand, O is smaller than O2- because the shielding effect of the
electrons in O2- is higher than that in O, therefore the effective nuclear charge in O 2-
is weaker than in O that is why O2- is bigger than O.
C. Ionization Energy – energy necessary to remove electrons from an atom.

~ 38 ~
Ao + energy → A+ + e-
Trend: Ionization energy increases across a period and as one goes up a
group in the periodic table.
Reason: Metals easily lose electrons because they are electron donors. Non-
metals do not readily lose electrons because they are electron acceptors.
Therefore, lower amount of energy is just required to remove electrons from a metal
but higher amount of energy is required to remove electrons in the non – metal part
of the periodic table.
Across a period, the shielding effect is lower, and effective nuclear charge is
stronger, therefore, outer electrons are strongly attracted by the nucleus,
consequently, a higher energy is necessary to remove outer electron. On the other
hand, as one goes up a period, atomic size decreases, therefore, the stronger the
attractive force between the nucleus and the outer electrons, consequently, these
electrons require higher amount of energy to be removed from their energy level.
Example: Cl has higher ionization energy than S, but F has higher ionization energy
than Cl or F > Cl > S in terms of ionization energy.
D. Electronegativity – the tendency of the atom to attract electron towards itself.
Trend: Electronegativity increases across a period and as one goes up a group
in the periodic table.

Reason: Metals are electron donors. Therefore, they have little tendency to
attract electron while non – metals are electron acceptors, therefore, they
greatly attract electrons to fulfill the configuration of their nearest noble gas to
achieve stability.

Example: Arrange the following elements according to increasing


electronegativity: Se, Te, Sb, As
Answer: Sb < Te < As < Se

E. Electron Affinity – the amount of energy released when a neutral atom accepts
and electron.
Ao + e- → A- + energy
Trend: Electron affinity increases across a period and as one goes up a group in
the periodic table.
Reason: Metals are electron donors. Therefore, they have little chance of
releasing energy because they have little chance of accepting electrons. Non
– metals are electron donors, therefore they have great chances of attracting
electrons, releasing high amounts of energy.
Example: Arrange the following elements according to decreasing electron
affinity: K, Na, Rb, Cs
Answer: Na > K > Rb > Cs

Activity 8
1. Arrange the following elements according to increasing atomic size: Ne, F, F-,
Na, Na+
2. Arrange the following elements according to decreasing metallic character:
S, O, F, Cl
3. Given the following elements: K, Br, Sr, Te
a. Arrange according to decreasing ionization energy.
b. Arrange according to increasing electronegativity.
c. Arrange according to decreasing electron affinity.
~ 39 ~
 Electronic Structure of atoms

The electrons of the atom are circling around the nucleus and can be found
most of the time in the atomic orbitals. Each sublevel or sub-shell contains
specific number of orbitals. There is just 1 orbital in the s (sharp) sub-level. There
are 3 orbitals present in the p (principal) sublevel. The d (diffuse) sublevel
contains 5 orbitals and f (fundamental) sublevel has 7 orbitals. Each sublevel can
accommodate only certain number of electrons. The s sublevel can
accommodate ≤ 2 electrons, p can accommodate ≤ 6 electrons, d sublevel can
accommodate ≤ 10 electrons and f sublevel can accommodate ≤ 14 electrons.
With the present number of elements in the periodic table, there could only be 7
main energy level available. The names of these main energy levels are K shell or
n=1, L shell or n=2, M shell or n=3, N shell or n=4, O shell or n=5, P shell or n=6 and
Q shell or n=7. The n=1 or the K shell is the closest energy level to the nucleus and
the n=7 or the Q shell is farthest the nucleus. The higher the value of n, the higher
the available energy level for the electrons.

You have to remember:

The nearest energy level to the nucleus contains


the least amount of energy, the farthest energy level has
the highest amount of energy, the nearest electrons to
Distribution
the nucleus of electrons in the stable
are the MOST different sublevelsthe
electrons, in the main energy level (mel)
farthest
electrons from the nucleus are the LEAST stable electrons
in the atom.

~ 40 ~
Problem 1. What is the maximum number of orbital and electrons present in n=5?
Answer:
For number of orbital:
No. of orbital = n2 No. of electrons = 2(n)2
= (5)2 = 2(5)2
= 25 = 50

 Electron Configuration
The distribution of electrons among the various orbitals in an atom or molecule is
known as electron configuration. The principle that governs the manner of
distribution of electrons is Aufbau Principle which is also known as ―building up‖
principle. Aufbau principle states that in the ground state of an atom or ion,
electrons fill atomic orbitals of the lowest available energy levels before occupying
higher levels.
In terms of energy level:
You have to remember:
In terms of energy level:
n=1 < n=2 < n=3 …
s<p<d<f
In the case of the sublevels p, d and f where they have more than 1 orbitals, each
of their orbitals has the same energy level. They are referred to as degenerate
orbital – orbital that have the same energy level. For each orbital, Wolfgang Pauli in
his Pauli’s Exclusion Principle posited that ―only two electrons can be
accommodated in each orbital‖ He further stated that the ―electrons occupying
the same orbital should be in opposite spin.‖ Likewise, he emphasized that ―no two
electrons of the same atom will have the same four sets of quantum numbers.‖
To guide you in writing the electron configuration of an atom, you can use the
following mnemonic device:

~ 41 ~
Study and understand the following illustrations:
1. The electron configuration of is 1s2 2s2sp6 3s23p3
2. The electron configuration of is
1s 2s sp6 3s23p6 4s23d6
2 2

3. The electronic configuration of Aluminum ion 3+

Solution: The atomic number of Al is 13 and it caries a charge of 3+, therefore


the ion has lose 3 electrons, so its electron number is equal to 10. Its electronic
configuration is 1s2 2s22p6
Activity 9
Write the electronic configuration of the following atoms and ions:
1. 4. 4+

2. 5. -

3.
You can also write the electronic configuration of atoms by using the electronic
configuration of the nearest noble gas of the element.
Example 1: What is the electronic configuration of ?
Solution: The nearest noble gas to Ag is Kr. Its atomic number is 36. Its electron
configuration is 1s2 2s22p6 3s23p6 4s23d104p6. We use this configuration of Kr to write
the abbreviated electronic configuration of 47 Ag. Therefore: [Kr] 5s24d9
Example 2. What is the abbreviated electronic configuration of ?
Answer: [Xe] 6s2 4f145d106p3

Activity 10
Write the electronic configuration of the following elements using the electronic
configuration of their nearest noble gas:
1. 4. 2+

2. 5. 3-

3.

Consider the electronic configuration of the following atoms and ions: 15P3-, 16S2-,
17Cl-, 18Ar, 19K+, 20Ca2+, 21Sc3+. What did you observe on their electronic
configuration?
a. 15P3- - 1s2 2s22p6 3s23p6
b. 16S2- - 1s2 2s22p6 3s23p6
c. 17Cl- - 1s2 2s22p6 3s23p6
d. 18Ar - 1s2 2s22p6 3s23p6
e. 20Ca2+ - 1s2 2s22p6 3s23p6
f. 21Sc3+ - 1s2 2s22p6 3s23p6

~ 42 ~
Answer: They all have the same electronic configuration.
Substances that have similar electronic configuration are termed isoelectronic.
Neutral atoms tend to either lose or gain electrons in order to achieve the
configuration of the noble gases. The configuration of the noble gas is very unusual
because it has completely filled outermost energy level giving the noble gas an
unusual stability. This is the status that every atom or ion wants to achieve and can
be done by either losing or gaining of electrons.

Activity 11
Write the electronic configuration of the following atoms, ions and molecules and
group the substances that are isolectronic:
Mg, F-, Si, H2O, NH3, O2, CO

 Excited and Ground State Configuration


The electronic configuration of the elements which are reflected on the periodic
table of elements is in the ground state – condition whereby electrons in the atom
occupy the lowest energy level. However, when atoms are exposed to extreme
heat and electricity, their electrons are in the excited state – condition when
electrons jumped into a higher energy level.
Example: The ground state electronic configuration of Oxygen atom is 1s2 2s22p4.
What are the electronic configurations of Oxygen in the excited state?
Answer: a. It can be 1s2 2s12p5. It is shown here that 1 electron in the 2s orbital is
promoted to 2p orbital. 2p orbital has higher energy level that 2s orbital.
b. It can also be 1s2 2s22p3 3s1. One electron in the 2p orbital has jumped to 3s
orbital. 3s orbital has higher energy level that 2p orbital.

 Distribution of Electron Using Orbital Box Diagram

Electrons can be distributed using orbital box diagram. Each box represents an
orbital and contains 2 electrons that have opposite spins (Pauli‘s Exclusion
Principle). For sublevels with more than 1 orbital (degenerate orbital), the
distribution of electrons in the different orbitals follow the Hund’s Rule of Maximum
Multiplicity – which states that electrons spread in the empty orbital of the same
energy level in the same spin before pairing up in parallel spin. This was proposed
by the German physicist Friedrich Hund.
Illustrations:

a.

b.

c.

d.

~ 43 ~
e.
The following are not allowed according to Hund‘s Rule:

a.

b.

c.

d.
The electronic configuration of an atom helps you predict the following:
a. Period where the element belongs. Example: The electron configuration of 11Na is
1s2 2s22p6 3s1, the highest energy level in the electron configuration is 3, therefore,
Na belongs to period 3.
b. Group of the element. Using the same example above, the valence electron of
Na is 1, therefore, it belongs to Group IA of 1.
c. If the element is metal, non-metal or metalloid. If the number of valence electron
is lesser than 4, the element is metal, but if the number of valence electron is more
than 4, the element is non – metal. Using the above example, the number of
valence electron of Na is 1, therefore, Na is a metal.

References:
Chang, R. Chemistry.(2010). 5th ed. Mc Graw Hill Book Co.. N.Y.

Manahan, Stanley E. (1993) Fundamentals of Chemistry. London: Lewis.

Masterton and Hurley. (2004). Chemistry: Principles and Reactions. 5th ed. Thomson
Learning, Inc.

Purdue University. (22002). Special Edition of General Chemistry. McGraw-Hill Primis


Custom Publishing.
https://www.livescience.com/25300-periodic-
table.html#:~:text=The%20periodic%20table%20of%20elements,coincides%20with%2
0increasing%20atomic%20mass.
https://en.wikipedia.org/wiki/Timeline_of_chemical_element_discoveries#:~:text=C
hemistry%20Portal&text=The%20discovery%20of%20the%20118,elements%20cannot
%20be%20accurately%20determined.
https://www.sciencelearn.org.nz/resources/1804-development-of-the-periodic-
table
https://chem.libretexts.org/Bookshelves/General_Chemistry/Map%3A_Chemistry_-
_The_Central_Science_(Brown_et_al.)/07._Periodic_Properties_of_the_Elements/7.1%
3A_Development_of_the_Periodic_Table

~ 44 ~
https://chem.libretexts.org/Courses/Valley_City_State_University/Chem_121/Chapte
r_2%3A_Atomic_Structure/2.03%3A_Families_and_Periods_of_the_Periodic_Table
http://www.chem.uiuc.edu/rogers/Text3/Tx33/tx33.html
http://www.knowledgedoor.com/2/elements_handbook/origin_of_element_name.
html
https://www.thoughtco.com/element-groups-vs-periods-
608798#:~:text=Groups%20and%20periods%20are%20two,group%20or%20across%20
a%20period.
https://www.google.com/search?q=periodic+trends&rlz=1C1RLNS_enPH872PH872&
oq=periodic+trends&aqs=chrome.0.69i59j0l7.3881j0j15&sourceid=chrome&ie=UTF-8
https://courses.lumenlearning.com/introchem/chapter/the-shielding-effect-and-
effective-nuclear-charge/
https://www.chem.tamu.edu/class/fyp/stone/tutorialnotefiles/fundamentals/trends.
htm#:~:text=WHY%3F%20%2D%20The%20number%20of%20energy,a%20period%2C%
20atomic%20radius%20decreases.
https://www.google.com/search?q=ground+state+configuration&rlz=1C1RLNS_enP
H872PH872&oq=ground+state+configuration&aqs=chrome..69i57j0l7.7347j0j15&sour
ceid=chrome&ie=UTF-8
https://www.chem.fsu.edu/chemlab/chm1045/e_config.html
https://byjus.com/chemistry/electron-configuration/

~ 45 ~
Lesson 4: CHEMICAL BONDING

Introduction

This year, man has been startled with the pandemic caused by CoVid – 19 virus.
Everybody has been challenged to undertake protective measures to prevent the
spread of the virus. Various chemical substances produced through chemical
technology became an in-demand commodity for the public. The supply of
antiseptics like alcohol, sanitizers, bath soaps and medicine like vitamins to boost
the immune system have abruptly decreased causing public alarm. These various
chemical substances are produced from chemical interaction of atoms and the
individual atoms in these compounds are held together by chemical bonds.

Learning Outcomes
At the end of the lesson, you should be able to:
 differentiate the properties of the different kinds of bonds;
 illustrate the Lewis dot structure of compounds;
 write and name the chemical formula of the compounds;
 calculate the oxidation number used by an element in the compound;
 determine the empirical and molecular formula of the compound.

Activating Prior Learning

Watch the Music video of the song ―Share the Love‖ by Daniel Padilla, Kathryn
Bernardo, Elmo Magalona and Janella Salvador
(https://www.youtube.com/watch?v=Oeu7W-fTxmk) and make reflection on the
message of the song.
_________________________________________________________________________________
_________________________________________________________________________________
_________________________________________________________________________________

Presentation of Content

Every atom of a substance is interacting with other atoms to achieve stability. The
valence electrons are responsible to achieve this goal during chemical interaction
to form various compounds. The atoms in the compounds are held together by a
strong attractive force called chemical bonds.
There are three different types of bonds entered into by atoms during compound
formation. These are ionic bond, covalent bond and metallic bond.

Ionic Bond

~ 46 ~
Ionic bond is the result of the transfer of electrons from one substance (metal –
electron donor) to another substance (non – metal – electron acceptor). When a
metal loses an electron, it yields a positive charge (cation), and when non – metal
gains an electron, it yields negative charge (anion). These oppositely charge
particles will then attract each other and eventually form a compound. In this case,
because the bond is formed from opposite charges, ionic bond is also known as
electrovalent bond. The compound formed from the bonding of a metal and non –
metal is known as ionic compounds.

Examples:
1. The bond between 11Na and 17Cl. Na is a metal (electron donor) and Cl is a
non – metal (electron acceptor). The electron configuration of 11Na is 1s2
2s22p6 3s1. It has valence electron equal to 1 and therefore, can lose 1
electron to become Na+ to have 8 electrons in its outermost energy level
(octet rule). On the other hand, 17Cl has electron configuration of 1s2 2s22p6
3s23p5. It has valence electrons equal to 7 and should gain 1 electron to
become Cl- to gain stability according to octet rule.

Note: Because Na lost only 1 e- and Cl gained only 1 e-, therefore, the atomic ratio
of the compound formed is 1:1, that is why the formula of the compound
formed is NaCl.

2. Formation of the compound of Mg and I


Mg belongs to IIA, therefore it has valence electrons equal to 2 and can lose
2 e- to achieve stability. I belongs to VIIA, therefore, its valence electron is
equal to 7 and needs 1 e- to have 8 e- in its outer energy level to achieve
stability.

Note: 1 atom of Mg can lose 2 e- and 1 atom of I can gain 1 e-, therefore, it needs 2
atoms of I to gain the 2 e- lost by Mg to form MgI2.

3. Formation of K and O compound

Note: Oxygen belongs to Group VIA, it has 6 valence electrons and needs 2 more
electrons to be stable according to octet rule. On the other hand, K belongs
to IA and has 1 valence electron. It can donate 1 electron to become stable.

~ 47 ~
So, it needs 2 atoms of K to donate the number of electrons required by O to
become stable. So therefore, the formula of the compound is K2O.

4. Formation of Al and S compound

Note: Al belongs to family IIIA and has 3 ve-, therefore it can donate 3 e- to achieve
a charge of 3+ to become stable. On the other hand, S belongs to family VIA
and has 6 ve-, so it needs 2 e- to have 8 e- in its outermost energy level.
Therefore, 2 atoms of Al is required to fulfill the requirements for electrons of 3
atom of S to achieve stability. So the formula of its compound is Al 2S3.

Another way of predicting the formula of the compound is criss – crossing method.
This is accomplished by exchanging the charge of the atoms and becomes
subscript in its formula.(Note: The charges of each element is reflected on the
periodic table.
Example:
1. Formula of the compound of Na and Cl

Note: If the charge is 1, you don‘t need to write it as subscript in the chemical
formula of the compound.

2. Formula of the compound of Mg and I

3. Formula of the compound of K and O

4. Formula of the compound of Ca and S

Note: If the subscript is divisible with each other, reduce it to lowest term.

5. Formula of the compound of Ba and N

~ 48 ~
Covalent Bond
Covalent bond is the result of sharing of electrons by 2 or more non – metals to
achieve stability. The compound formed from this bonding type is known as
covalent compound. Because covalent compound is composed of both non –
metals, the compound is classified as molecule. Covalent bonds may classified as
single bond ( - ), double bond ( = ) or triple bond ( ≡ ). Single bond consists of 2
shared electrons, double bond consists of 4 shared electrons and triple bond
consists of 6 shared electrons. The following are examples of covalent bonding.
1. Bond formed between O and F

Note: Based on the structure, the formula of the molecule is OF 2

2. Bond formed between H and O to form water molecule

Note: H belongs to Group IA of the periodic table, therefore, it has 1 valence


electron and is expected to share 1 electron to become stable. H is the only
element in group IA that is capable of sharing electrons to other non –
metals to achieve stability. O on the other hand, belongs to group VIA and
has 6 valence electrons and needs 2 electrons to have 8 electrons in its
outermost energy level. Its molecular formula is H2O.

3. Bond formed between N and H

Therefore, the molecular formula formed form the bonding of N and H is NH 3.

4. Compound formed by C and O

5. Compound formed by S and O

~ 49 ~
or

Note: Based on the above structure of SO2, any of these structures can be a correct
Lewis Dot structure of SO2. Each of these structures is called resonance
structure of SO2. Resonance structure is one of two or more Lewis structures for
a single molecule that cannot be represented accurately by only one Lewis
structure. The double-headed arrow indicates that the structures shown are
resonance structures. The term resonance itself means the use of two or more
Lewis structures to represent a particular molecule.

6. Compound formed by H, C and N

Metallic Bond
The chemical bonding that holds the bulk of atoms of a metal together is called
metallic bond. Metallic bonds are formed from the attraction between mobile
electrons (sea of electrons) and fixed, positively charged metallic atoms. The
unusual strength and durability of metals together with some of its physical
properties like conductivity, malleability, ductility, luster, melting point and heat
capacity are explained by metallic bonding that occurs among its atoms. The
copper used in electrical wirings and aluminum foil are just few of the examples
that demonstrate metaling bonding in metals. Another application of metallic
bonding is the production of alloys. Alloy is the combination of 2 or more metals
and is performed to improve the quality of metals. The following illustrates metallic
bonding based on Paul Drüde "sea of electrons" metallic bonding theory. To
understand better the mechanism of metallic bonding, watch the simulation at
https://www.youtube.com/watch?v=Bi0rUNV8mEw

~ 50 ~
The Electron Sea Model: Positive atomic nuclei (orange circles) surrounded by a sea
of delocalized electrons (yellow circles).

Lewis Dot Structure


Lewis structure is a representation of covalent bonding in which shared electron
pairs are shown either as lines or as pairs of dots between two atoms, and lone pairs
are shown as pairs of dots on individual atoms. Only valence electrons are shown in
a Lewis structure. It is composed of the symbol of the element which represent its
nucleus, dots which represent the number of valence electron of the atom and
dash which represent the bonded electrons in the molecule. A dash (-) is
equivalent 2 shared electrons, (=) is equivalent to 4 shared electrons and (≡) is
equivalent to 6 shared electrons. A pair of electrons that is not involved in bonding
is called a lone pair. Here are the basic steps employed in writing Lewis structure of
a molecule.
1. Write the skeletal structure of the compound, using chemical symbols and
placing bonded atoms next to one another. For simple compounds, this task
is fairly easy. For more complex compounds, we must either be given the
information or make an intelligent guess about it. In general, the least
electronegative atom occupies the central position. Hydrogen and fluorine
usually occupy the terminal (end) positions in the Lewis structure.
2. Count the total number of valence electrons present.
a. For a neutral compound like H2O, the total number of valence electrons is
8. (For H, H has 1 valence electron and there are 2 atoms of H in the
formula H2O, so, 2 x 1 = 2. For O, there are 6 valence electron of O and
there is just 1 atom of O in the formula H2O, so, 6 x 1 = 6. Adding 2 to 6
equals 8).
b. For polyatomic anions, add the number of negative charges to that total.
(For example, for the CO32- ion we add two electrons because the 2-
charge indicates that there are two more electrons than are provided by
the atoms. For C, 4 x 1 =4. For O, 6 x 3 = 18. Adding 4 to 18 equals 22 but
add 2 here because of the charge 2-. Therefore, for the entire CO3 2-, it
has 24 valence electrons.)
c. For polyatomic cations, we subtract the number of positive charges from
this total. (Thus, for NH4+ we subtract one electron because the 1+ charge
indicates a loss of one electron from the group of atoms. For N, 1 x 5 = 5.
~ 51 ~
For H, 1 x 4 = 4. Adding 5 to 4 gives 9 but subtract 1 because of the 1+
charge in NH4+. Therefore total valence electron in NH4+ is 8.)
3. Draw a single covalent bond between the central atom and each of the
surrounding atoms. Complete the octets of the atoms bonded to the central
atom. (Remember that the valence shell of a hydrogen atom is complete
with only two electrons.) Electrons belonging to the central or surrounding
atoms must be shown as lone pairs if they are not involved in bonding. The
total number of electrons to be used is that determined in step 2.
4. After completing steps 1–3, if the central atom has fewer than eight electrons,
try adding double or triple bonds between the surrounding atoms and the
central atom, using lone pairs from the surrounding atoms to complete the
octet of the central atom.

Illustrative examples:
Example 1. Illustrate the Lewis structure for H2O molecule.
H O H
Based on rule #1, H occupies the terminal position.
1. The total number of valence electron of the molecule is 8. H has 2 atoms
in the molecule and its valence electron is 1. So 2 x 1 = 2. For O, there is just
1 atom of O in the molecule and it has 6 valence electrons. So I x 6 = 6.
Add up the valence electrons, 2 = 6 = 8.

2. Connect central atom with the attached atoms using single bonds.

3. Check whether octet rule is obeyed by counting the number of valence


electrons around the atom. Note: H is stable at having 2 valence electrons
only in a molecule.

Note: If each atom in the molecule obeys the octet rule, the Lewis structure is
acceptable. For H2O, H obeys octet rule by having 2 e-. Therefore, the acceptable
Lewis structure for H2O is:

Example 2. Illustrate the Lewis structure of CO

~ 52 ~
Therefore, the acceptable Lewis structure of CO is:

Example 3. Illustrate the Lewis structure of CH3OH

Example 4. Illustrate the Lewis structure of C2H4.

Naming Chemical Formula


A chemical formula is an expression showing the chemical composition of a
compound in terms of the symbols for the atoms of the elements involved. If the
chemical formula of the compound constitutes 2 different atoms, the compound is
classified as binary compound. On the other hand, if the formula constitutes three
or more different atoms, the compound is classified as ternary compound.

Naming Ionic Compounds


A. Binary Compound – compound composed of a metal and a non – metal.
Rule: Name the metal followed by the name of the non – metal with -ide ending.
Name of non – metals
Carbon – carbide fluorine – fluoride
Silicon – silicide chlorine – chloride
Nitrogen – nitride iodine – iodide
Oxygen – oxide bromine – bromide
Phosphorus – phosphide Tellurium – telluride
Sulfur – sulfide hydrogen – hydride

Illustrative examples:
What is the name of the following chemical formula of binary compounds:
Chemical Name of Name of the Non – metal with Name of the formula
Formula the metal -ide ending
NaCl sodium chloride Sodium chloride
K2O potassium oxide Potassium oxide
MgS magnesium sulfide Magnesium sulfide
Ca3N2 Calcium nitride Calcium nitride
AlBr3 Aluminum bromide Aluminum bromide
Ba3P2 Barium Phosphide Barium phosphide

~ 53 ~
B. Ternary Compounds – combination of a metal with a polyatomic ion
Rule: Name the metal followed by the name of the polyatomic ion.
Illustrative examples:
What is the name of the following chemical formula of ternary compounds:
Chemical Name of the
Name of the metal Name uof the formula
formula polyatomic ion
Na2SO4 sodium sulfate Sodium sulfate
K2C2O4 potassium oxalate Potassium oxalate
Al(OH)3 aluminum hydroxide Aluminum hydroxide
Ca3(AsO4)2 calcium arsenate Calcium arsenate
LiNO2 lithium nitrite Lithium nitrite
Mg(C2H3O2)2 magnesium acetate Magnesium acetate
RbClO4 rubidium Perchlorate Rubidium perchlorate

List of ions
A. Cation
Name Formula Name Formula
ammonium NH4+ Lead (II) or plumbous Pb 2+
Hydronium H3O+ Lead (IV) or plumbic Pb4+
Copper (I) or cuprous Cu+ Mercury (I) or mercurous Hg+
Copper (II) or cupric Cu2+ Mercury (II) or mercuric Hg2+
Cobalt(II) or cobaltous Co2+ Nickel (II) or nickelous Ni2+
Cobalt (III) cobaltic Co3+ Nickel (III) or nickelic Ni3+
Iron (II) or ferrous Fe2+ Tin (II) or stannous Sn2+
Iron (III) or ferric Fe3+ Tin (IV) or stannic Sn4+

B. Anions
Name Formula Name Formula
acetate C2H3O2 - metaborate BO3 -
aluminate AlO2 - metaphosphate PO3 -
argenticyanide Ag(CN)2 - metasilicate SiO3 2-
arsenate AsO4 3- molybdate MoO4 2-
arsenite AsO3 3- nitrate NO3 -
benzoate C7H5O2 - nitrite NO2 -
bicarbonate HCO3 - orthosilicate SiO4 4-
bismuthate BiO4 3- oxalate C2O4 2-
bisulfate HSO4 - perchlorate ClO4 -
bisulfite HSO3 - periodate IO4 -
Borate BO3 3- permanganate MnO4 -
bromate BrO3 - peroxide O2 2-
carbonate CO3 2- phosphate PO4 3-
chlorate ClO3 - phosphite PO3 3-
chlorite ClO2 - plumbite PbO2 2-
chromate CrO4 2- pyroarsenate As2O7 4-
cyanate CNO- pyrophosphate P2O7 4-
cyanide CN- silicate SiO3 2-

~ 54 ~
dichromate Cr2O7 2- stannate SnO3 2-
ferricyanide Fe(CN)6 3- stannite SnO2 2-
ferrocyanide Fe(CN)6 4- sulfate SO4 2-
formate HCOO- sulfite SO3 2-
hydroxide OH- tartrate C4H4O6 2-
hypobromite BrO- tetraborate B4O7 2-
hypochlorite ClO - tetrathionate S4O6 2-
iodate IO3 - thiocyanate SCN-
iodite IO2 - thiosulfate S2O3 2-
manganate MnO4 2- Zincate ZnO2 2-
metaarsenite AsO2 -

Writing chemical formula of a ternary compound

The formula of a ternary compound can be obtained using a criss – cross method.
This involves the exchange of charges by the cation and the anion component of
the compound which then become the subscript in the formula of the compound.

Illustrative examples:

1. The formula of the compound formed from Na+ and OH-

Note: if the subscript in the formula is 1, you don‘t need to write it on the formula.

2. Formula formed by Ca2+ and NO3 -

Note: During criss – crossing, the formula of the anion should never change. Use
parenthesis to separate subscript 3 from the anion NO3 and 2, the subscript that
applies to the entire NO3 molecule.

3. Formula formed by Al 3+ and SO4 2-

4. Formula formed by Magnesium ion and dichromate ion

5. Formula formed by Barium ion and ferrocyanide ion

~ 55 ~
Naming Formula of a ternary compound

Rule: Name the metal (cation) followed by the name of the polyatomic ion (anion).

Illustrative examples:

Formula Name of Name of Name of the


cation anion(polyatomic formula
(metal) ion)
NaOH sodium hydroxide Sodium
hydroxide
Ca(NO3)2 calcium nitrate Calcium
nitrate
Al 2(SO4)3 aluminum sulfate Aluminum
sulfate
MgCr2O7 magnesium dichromate Magnesium
dichromate
Ba2Fe(CN)6 barium ferrocyanide Barium
ferrocyanide

Activity 1

Write the formula formed by the following ions and name the formula of the
compound.

Formula
anion
cation NO2- PO33- AsO43- ClO- PbO22- ZnO22-
Ag+
Name
Zn2+
Name
~ 56 ~
Hg2+
Name
Ni3+
Name
Si4+
Name
Fe3+
Name

Naming compounds of metals with multiple charges or oxidation state or number

Look at the charge or oxidation number of some of the metals found at the
transition state of the periodic table, they have multiple charge or oxidation
number. There are two systems of naming these compounds, the classical -ous and
-ic ending and stock system or the systematic or the IUPAC system. In the classical -
ous and -ic ending, -ous ending is given to the compound with the metal having
lower oxidation number while -ic ending is given to the metal having higher
oxidation number. On the other hand, in the stock system or systematic or IUPAC,
the charge of the metal is indicated by the Roman mineral enclosed with a
parenthesis.

Illustrative examples
Metal Formula
with
multiple
oxidation
Number
catio anion
O2- CO32- SO42-
Fe2+ FeO FeCO3 FeSO4
Name Ferrous oxide Ferrous carbonate Ferrous sulfate
Iron (II) oxide Iron (II) carbonate Iron (II) sulfate
Fe3+ Fe2O3 Fe2(CO3)3 Fe2(SO4)3
Name Ferric oxide Ferric carbonate Ferric sulfate
Iron (III) oxide Iron (III) carbonate Iron (III) sulfate
Cu+ Cu2O CuCO3 Cu2SO4
Name Cuprous oxide Cuprous carbonate Cuprous sulfate
Copper (I) oxide Copper (I) carbonate Copper (I) sulfate
Cu2+ CuO CuCO3 CuSO4
Name Cupric oxide Cupric carbonate Cupric sulfate
Copper (II) oxide Copper (II) carbonate Copper (II) sulfate
Pb2+ PbO PbCO3 PbSO4
Name Plumbous oxide Plumbous carbonate Plumbous sulfate
Lead (II) oxide Lead (II) carbonate Lead (II) sulfate
Pb4+ PbO2 Pb(CO3)2 Pb(SO4)2
Name Plumbic oxide Plumbic carbonate Plumbic sulfate
Lead (IV) oxide Lead (IV) carbonate Lead (IV) sulfate

Naming Covalent compounds

~ 57 ~
Covalent compounds are compounds formed from the bonding of 2 or more non –
metals.

Rule: Name the first element of the formula (element with lower electronegativity)
followed by the name of the second element with – ide ending. If subscript is
present in the formula, use the following prefix to indicate the number of atoms in
the formula.

1 – mono 6 – hexa

2 – di 7 – hepta

3 – tri 8 – octa

4 – tetra 9 – nona

5 – penta 10 – deca

Note: If there is just 1 atom in the first element of the covalent compound, there is
no need to use the prefix mono. For oxides, the ending ―a‖ in the prefix is sometimes
omitted. For example, N2O4 may be called dinitrogen tetroxide rather than
dinitrogen tetraoxide.

Illustrative examples

CO – carbon monoxide N2O4 – dinitrogen tetroxide

CO2 – carbon dioxide SO2 – sulfur dioxide

CCl 4 – carbon tetrachloride P2O5 – diphosphorus pentoxide

OF2 – oxygen difluoride P4O10 – tetraphosphorus decoxide

SF6 – sulfur hexafluoride CS2 – carbon disulfide

Note: There are some covalent compounds that are give special names known as
common name. Examples are H2O for water and NH3 for ammonia. The IUPAC
name of these compounds are dihydrogen monoxide for H2O and nitrogen
trihydride for NH3.

Naming acids

Acids are compounds that can change blue litmus paper to red, has sour taste and
can give burning sensation. It is a substance that increases the hydronium ion
(H3O+) concentration in water solution. The presence of the atom Hydrogen (H) in
front of the formula is an indication that the compound is acid. There are 2 types of
acids. The binary acid and the oxoacid. Binary acid just contains 2 elements and
should contain the symbol (aq) as subscript, which mean aqueous, in the formula.
Oxoacids contain the Hydrogen and oxy anion. Oxyanion is any polyatomic anion

~ 58 ~
that contains Oxygen. Examples include, NO3 -, NO2 –, PO4 3-, PO3 3-, SO4 2-, SO3 2-

etc.

A. Naming Binary acid – Hydrogen + non – metal

Rule: Name hydrogen as ―hydro‖ followed by the name of the attached non –
metal with -ic ending and acid as last part of its name.

Examples:

H2S(aq) – hydrosulfuric acid H3P(aq) – hydrophosphoric acid

H3N(aq) – hydronitric acid HF(aq) – hydrofluoric acid

H2C(aq) – hydrocarbonic acid HCl (aq) – hydrochloric acid

B. Naming oxoacids – Hydrogen + oxyanion

Rule: Name the oxyanion that is present in the compound. If the name ends with -
ate, change it to -ic ending, and if the name of the oxyacid ends with -ite, change
it to -ous ending. End the name with the word acid.

Illustrative examples

Formula Name of
oxyanion
H2CO3 carbonate Carbonic acid

H2SO3 sulfite Sulfurous acid

H3 AsO4 arsenate Arsenic acid

H2C2O4 oxalate Oxalic acid

HC2H3O2 acetate Acetic acid


H2PbO2 plumbite Plumbous acid

Determination of oxidation number

Oxidation number, also called oxidation state refers to the total number of electrons
that an atom either gains or loses in order to form a chemical bond with another
atom.

Rule in determining the oxidation number used by an element in a compound:

1. Assign the oxidation number of each element in the compound.


a. All elements belonging to 1A has 1+, IIA has 2+, IIIA has 3+, VA has 3-, VIA
has 2- and VIIA has 1- oxidation number.
b. Oxygen is always 2- EXCEPT in peroxide where it takes 1- oxidation
number.

~ 59 ~
c. Hydrogen is always 1+ EXCEPT for hydride where it takes 1- oxidation
number.
2. The algebraic sum of the oxidation number of a neutral compound is always
equal to zero. A neutral compound is any compound that does not carry a
charge.
3. The algebraic sum of the oxidation number of polyatomic ion or a radical is
equal to its net charge. A radical is a group of atoms that carry a charge.
Examples are PO3 3-, CrO4 2-, IO3 -, MnO4 2-, NO3 -, SO4 2-, etc.
Illustrative examples:

1. What is the oxidation number used by S in H2SO4?

2. What is the oxidation number used by C in ammonium acetate, NH 4C2H3O2?

3. What is the oxidation number of arsenic in Magnesium arsenate?

~ 60 ~
4. What is the oxidation number used by Cr in Cr2O7 2-?

5. What is the oxidation number of Carbon in ferrocyanide ion?

Note: The charge of Fe is 2+ because the ion is ferrocyanide. If the ion is


ferricyanide, it should be 3+.

Activity: Determine the oxidation number of the underlined symbol of the element
in the compound. Show complete solution

1. Na3PO3
2. Al 2(MoO4)3
3. Pb(NO3)4
4. Fe3(BiO4)2
5. Zn(ClO2)2
6. C4H4O6 2-
~ 61 ~
7. Determine the oxidation number of Arsenic in calcium pyroarsenate.

Determination of Empirical and Molecular formula

Empirical formula is the simplest formula of a compound while molecular formula is


the actual formula for a molecule. Always remember that all empirical formula can
be a molecular formula but not all molecular formula can be an empirical formula.

Step in Determining Empirical Formula

1. Determine the mass of the individual component of the compound. (Note: if


percent composition is given, convert it directly to mass in grams.)
2. Determine the number of moles of the individual component of the
compound.

3. Get the mole ratio by dividing the individual mole of the component with the
computed lowest number of moles among the components of the
compound.
4. The mole ratio becomes the simplest whole number ratio of the atoms in the
formula of the compound which is called the empirical formula.
Note: if the computed mole ratio is not a whole number, you may round -off to the
nearest whole number or multiply it with a number that gives you a product that is a
whole number. Example, if the decimal part of the computed mole ratio is 0.01 to
0.15 and 0.9 to 0.99, round – off to nearest whole number. But if the decimal part is
0.25, multiply it by 4, if 0.33, multiply it by 3 and if 0.5, multiply it by 2.

Illustrative examples

1. A sample of a compound contains 1.52 g of nitrogen (N) and 3.47 g of


oxygen (O). 1. What is the empirical formula of the compound? 2. What is the
empirical mass of the compound?

You may follow this route to arrive at empirical formula

1. a. Mass of the component


N = 1.52 g
O = 3.47 g

a. Mole – use the formula n =

For N: n= = 0.109 mol lower number of mole

~ 62 ~
For O: n= = 0.217 mole

b. Mole ratio:
For N: =1

For O: =2

Therefore the empirical formula is NO2

2. Empirical ass
N = 1 x 14 g = 14 g/mol
O = 2 x 16 g = 32 g/mol
46 g/mol empirical mass

Example 2. Determine the empirical formula and empirical mass of the compound
that contains 40.0% carbon, 6.7% hydrogen, and 53.3% oxygen.

C = 40.0% = 40.0 g n= = 3.33 mole Lowest


# of
mole
H = 6.7% = 6.7 g n= = 6.7 mole

O = 53.3% = 53.3 g n= = 3.33 mole

C= =1
Therefore, the Empirical
H= =2 Formula (EF) is CH2O

O= =1

C = 1 x 12 g/mol = 12 g/mol
Therefore, the Empirical
H = 2 x 1 g/mol = 2 g/mol Mass (EM) 30 g/mol
O = 1 x 16 g/mol = 16 g/mol
30 g/mole

Example 3. What is the empirical formula for a compound containing 26.57%


potassium, 35.36% chromium, and 38.07% oxygen?
K = 26.57% = 26.57 g n= = 0.681 mole

Cr = 35.67% = 35.67 g n= = 0.686 mole

O = 38.07% = 38.07 g n= = 2.379 mole

~ 63 ~
K= =1

Cr = = 1.007 = 1

O= = 3.49 = 3.5

Therefore: EF = KCrO3.5 but you can not round – off 3.5 but rather, multiple this by 2
to obtain a whole number.
Therefore, (KCrO3.5)2 becomes K2Cr2O7, the empirical formula of the compound.
Determination of Molecular Formula
Molecular formula or MF = (EF)
Example 1.
A major textile dye manufacturer developed a new yellow dye. The dye has a
percent composition of 75.95% C, 17.72% N, and 6.33% H by mass with a molecular
mass of about 240 g/mol. Determine the molecular formula of the dye.
C = 75.95% = 75.95 g n= = 6.329 mole

N = 17.72% = 17.72 g n= = 1.266 mole

H = 6.33% = 6.33 g n= = 6.33 mole

C= = 4.999 = 5.0
Therefore, the Empirical
N= = 1.0 Formula is C5NH5

H= = 5.0

C = 5 x 12 g/mol = 60 g/mol
Therefore, the Empirical
N = 1 x 14 g/mol = 14 g/mol Mass = 79 g/mol

H = 5 x 1 g/mol = 5 g/mol
79 g/mole
MF = (EF)
= (C5NH5) Therefore, the Molecular
Formula = C15N3H15
= 3 (C5NH5)

MF = C15N3H15

Example 2.

A gas sample is found to contain 39.10% carbon, 7.67% hydrogen, 26.11% oxygen,
16.82% phosphorus, and 10.30% fluorine. If the molecular mass is 184.1 g/mol, what is
the molecular formula?

~ 64 ~
C = 39.10% = 39.10 g n= = 3.26 mole

H = 7.67% = 7.67 g n= = 7.67 mole

O = 26.11% = 26.11 g n= = 1.63

P = 16.82% = 16.82 g n= = 0.54

F = 10.30% = 10.30 g n= = 0.54

C= = 6.04 = 6.0

H= = 14.2 = 14
Therefore, the Empirical
Formula is C6H14O3PF
O= = 3.02 = 3

P= =1

F= =1

C = 6 x 12 g/mol = 72 g/mol

H = 14 x 1 g/mol = 14 g/mol
Therefore, the Empirical
O = 3 x 16 g/mol = 48 g/mol Mass = 184 g/mol

P = 1 x 31 g/mol = 31 g/mol

F = 1 x 19 g/mol = 19 g/mol
184 g/mol

MF = (EF)
Therefore, the Molecular
= (C6H14O3PF) Formula = C6H14O3PF

= 1 (C6H14O3PF)

= C6H14O3PF

Activity: Solve the following problems. Show complete solution.

~ 65 ~
1. Peroxyacylnitrate (PAN) is one of the components of smog. It is a compound
of C, H, N, and O. If the compound contains 19.8 percent C, 2.50 percent H,
11.6 percent N and 66.1 percent O. a. What is the empirical formula?
b. What is its molecular formula given that its molecular mass is about 120
g/mol?

2. Allicin is the compound responsible for the characteristic smell of garlic. An


analysis of the compound gives the following percent composition by mass:
C = 44.4 %; H = 6.21 %; S = 39.5 %; O = 9.86 %. Calculate its empirical formula.
What is its molecular formula given that its molecular mass is about 162
g/mol?

3. Monosodium glutamate (MSG), a food-flavor enhancer, has been blamed


for ―Chinese restaurant syndrome,‖ the symptoms of which are headaches
and chest pains. MSG has the following composition by mass: 35.51 percent
C, 4.77 percent H, 37.85 percent O, 8.29 percent N, and 13.60 percent Na.
What is its molecular formula if its molecular mass is about 169 g/mol?

4. Carbohydrates are compounds containing carbon, hydrogen, and oxygen in


which the hydrogen to oxygen ratio is 2:1. A certain carbohydrate contains
40.0 percent carbon by mass. Calculate the empirical and molecular
formulas of the compound if the approximate molecular mass is 178 g/mol.

5. A certain metal M forms a bromide containing 53.79 percent Br by mass.


What is the chemical formula of the compound?

References

Chang, R. Chemistry.(2010). 5th ed. Mc Graw Hill Book Co.. N.Y.

Manahan, Stanley E. (1993) Fundamentals of Chemistry. London: Lewis.

Masterton and Hurley. (2004). Chemistry: Principles and Reactions. 5 th ed. Thomson
Learning, Inc.

Purdue University. (22002). Special Edition of General Chemistry. McGraw-Hill Primis


Custom Publishing.

https://chem.libretexts.org/Bookshelves/Physical_and_Theoretical_Chemistry_Textb
ook_Maps/Supplemental_Modules_(Physical_and_Theoretical_Chemistry)/Chemica
l_Bonding/Fundamentals_of_Chemical_Bonding/Metallic_Bonding

https://study.com/academy/lesson/what-is-a-metallic-bond-definition-properties-
examples.html

https://www.britannica.com/science/oxidation-number

~ 66 ~

You might also like